OFO 2018 Problema 10

Problemas que aparecen en el Archivo de Enunciados.
Avatar de Usuario
AgusBarreto

OFO - Medalla de Bronce-OFO 2015 OFO - Medalla de Bronce-OFO 2016 OFO - Jurado-OFO 2017 FOFO 7 años - Jurado-FOFO 7 años OFO - Jurado-OFO 2018
FOFO 8 años - Jurado-FOFO 8 años OFO - Jurado-OFO 2019 FOFO Pascua 2019 - Jurado-FOFO Pascua 2019 FOFO 9 años - Jurado-FOFO 9 años COFFEE - Jurado-COFFEE Matías Saucedo
OFO - Jurado-OFO 2020 FOFO Pascua 2020 - Jurado-FOFO Pascua 2020 COFFEE - Jurado-COFFEE Carolina González COFFEE - Jurado-COFFEE Ariel Zylber FOFO 10 años - Jurado-FOFO 10 años
OFO - Jurado-OFO 2021 FOFO 11 años - Jurado-FOFO 11 años OFO - Jurado-OFO 2022
Mensajes: 157
Registrado: Sab 15 Sep, 2012 6:28 pm
Medallas: 18
Nivel: Exolímpico
Ubicación: San Martín, Buenos Aires

OFO 2018 Problema 10

Mensaje sin leer por AgusBarreto »

Sea $f : \mathbb N \to \mathbb R_{>0}$ una función que satisface las siguientes dos condiciones:
  • $f(mn)=f(m)f(n)$ para cualesquiera $m,n \in \mathbb N$ coprimos (es decir, que su máximo común divisor sea igual a 1);
  • $f$ es creciente, es decir, $f(n+1) \geq f(n)$ para todo $n \in \mathbb N$.
Probar que existe $\alpha \geq 0$ tal que $f(n) = n^{\alpha}$ para todo $n \in \mathbb N$.
ACLARACIÓN: $\mathbb N$ y $\mathbb R_{>0}$ denotan a los enteros positivos y los reales positivos respectivamente.
Avatar de Usuario
AgusBarreto

OFO - Medalla de Bronce-OFO 2015 OFO - Medalla de Bronce-OFO 2016 OFO - Jurado-OFO 2017 FOFO 7 años - Jurado-FOFO 7 años OFO - Jurado-OFO 2018
FOFO 8 años - Jurado-FOFO 8 años OFO - Jurado-OFO 2019 FOFO Pascua 2019 - Jurado-FOFO Pascua 2019 FOFO 9 años - Jurado-FOFO 9 años COFFEE - Jurado-COFFEE Matías Saucedo
OFO - Jurado-OFO 2020 FOFO Pascua 2020 - Jurado-FOFO Pascua 2020 COFFEE - Jurado-COFFEE Carolina González COFFEE - Jurado-COFFEE Ariel Zylber FOFO 10 años - Jurado-FOFO 10 años
OFO - Jurado-OFO 2021 FOFO 11 años - Jurado-FOFO 11 años OFO - Jurado-OFO 2022
Mensajes: 157
Registrado: Sab 15 Sep, 2012 6:28 pm
Medallas: 18
Nivel: Exolímpico
Ubicación: San Martín, Buenos Aires

Re: OFO 2018 Problema 10

Mensaje sin leer por AgusBarreto »

Solución oficial:

Para tener en mente:
Spoiler: mostrar
Basta con probar que $f(n)^{\frac{1}{log(n)}}$ (donde $log$ es el logaritmo natural) es constante (y no negativo) para todo $n \in \mathbb{N}$, ya que si llamamos $C$ a este valor constante obtenemos que $f(n) = C^{log(n)} = n^{log(C)} = n^{\alpha}$ donde llamamos $\alpha = log(C) \geq 0$. Esta igualdad vale porque:

$C^{log(n)} = n^{log(C)} \iff C = n^{\frac{log(C)}{log(n)}}$

y por propiedadad de cambio de base del logaritmo vale que $n^{\frac{log(C)}{log(n)}} = n^{log_n(C)} = C$, y entonces la igualdad es cierta.
Ahora sí:
Spoiler: mostrar
En primer lugar, dado un entero cualquiera $a \geq 3$ definimos para cada $n \in \mathbb{N}$ las siguientes dos sucesiones:
  • $S_n=1+a+a^2+\cdots+a^n$

    De acá podemos deducir que, como $S_n$ y $a$ son coprimos, vale que $f(S_n)=f(1+aS_{n-1})\ge f(aS_{n-1})=f(a)f(S_{n-1})$. Repitiendo este proceso varias veces obtenemos que $f(S_n)\ge f(a)f(S_{n-1})\ge f(a)^2f(S_{n-2})\ge\cdots\ge f(a)^{n-1}f(1+a)\ge f(a)^n$

    Concluímos que $f(S_n)\ge f(a)^n$ para todo entero $a \geq 3$ y para todo $n \in \mathbb{N}$.
  • $D_n=a^n-S_{n-1}$

    De acá, como $D_n$ y $a$ son coprimos, vemos que se cumple $f(D_n)\le f(D_n+1)=f(aD_{n-1})=f(a)f(D_{n-1})$. Repitiendo este proceso varias veces obtenemos que $f(D_n)\le f(a)f(D_{n-1})\le f(a)^2f(D_{n-1})\le\cdots\le f(a)^{n-1}f(a-1)\le f(a)^n$

    Concluímos que $f(D_n)\le f(a)^n$ para todo entero $a \geq 3$ y para todo $n \in \mathbb{N}$.
Ahora notemos que $D_{n+2} \geq a^{n+1} \geq a^{n} \geq S_{n-1}$ ya que:
$D_{n+2} = a^{n+2} - S_{n+1} \geq a^{n+2} - a^{n+1} - S_n \geq a^{n+2} - 2 a^{n+1} \geq a^{n+1} \geq a^{n} \geq \dfrac{a^n - 1}{a-1} = S_{n-1}$
para todo entero $a \geq 3$.

Tomando $n= \left\lfloor{log_a(k)}\right\rfloor$ con $k \in \mathbb{N}$ arbitrario obtenemos que $a^{n+1} \geq k \geq a^n$, ya que $n+1 \geq log_a(k) \geq n$. Llamamos $m = \left\lfloor{log_a(k)}\right\rfloor$ para no confundirnos.

¿Para qué tomamos ese valor de $m$? observemos que ahora tenemos $D_{m+2} \geq a^{m+1} \geq k \geq a^m \geq S_{m-1}$ y por lo tanto ganamos esta excelente propiedad que vale para todo $k \in \mathbb{N}$ y todo $a \geq 3$:

$f(a)^{m+2} \geq f(D_{m+2}) \geq k \geq f(S_{m-1}) \geq f(a)^{m-1}$

como además $log_a(k)+2 \geq m+2$ y $m-1 \geq log_a(k) -2$, obtenemos que

$f(a)^{log_a(k)+2} \geq f(a)^{m+2} \geq k \geq f(a)^{m-1} \geq f(a)^{log_a(k)-2}$

para todo $a \geq 3$. Ahora tomando otro entero $b\geq 3$ y juntando las desigualdades de $a$ con las de $b$, obtenemos que $f(a)^{log_a(k)+2} \geq k \geq f(b)^{log_b(k)-2}$ para todo par de enteros $a, b \geq 3$. Como $log_a(k)+2=\frac{log(k)}{log(a)} + 2$ y $log_b(k)-2=\frac{log(k)}{log(b)}-2$ (siendo $log$ el logaritmo natural) tenemos:

$f(a)^{log_a(k)+2} \geq f(b)^{log_b(k)-2} \iff f(a)^{\frac{log(k)}{log(a)} + 2} \geq f(b)^{\frac{log(k)}{log(b)}-2} \iff f(a)^{\frac{1}{log(a)} + \frac{2}{log(k)} } \geq f(b)^{\frac{1}{log(b)}-\frac{2}{log(k)}}$


Probaremos que el hecho de que esto valga para todo $k \in \mathbb{N}$ implica que $f(a)^{\frac{1}{log(a)}} \geq f(b)^{\frac{1}{log(b)}}$ (para aquellos que estén familiarizados con la noción de límite, basta con hacer tender $k$ a infinito a ambos lados, para los que no, lean la siguiente cuenta horrible demostración):
Spoiler: mostrar
Para esta demostración es mejor cambiar de forma la desigualdad:

$f(a)^{\frac{log(k)}{log(a)} + 2} \geq f(b)^{\frac{log(k)}{log(b)}-2} \iff f(a)^{\frac{log(k)+2log(a)}{log(a)}} \geq f(b)^{\frac{log(k)+2log(b)}{log(b)}} \iff f(a)^{\frac{log(k)+2log(a)}{log(k)+2log(b)}} \geq f(b)^{\frac{log(a)}{log(b)}}$

Lo que queremos probar ahora es que $f(a) \geq f(b)^{\frac{log(a)}{log(b)}}$, que es equivalente a lo que dijimos antes. Para esto supongamos, a modo de contradicción, que existe una constante $c$ tal que $f(b)^{\frac{log(a)}{log(b)}} > c \geq f(a)$. Veamos si podemos tomar $k$ de modo que $f(a)^{\frac{log(k)+2log(a)}{log(k)+2log(b)}}$ sea menor que $c$ y así llegar a un absurdo. Planteemos la desigualdad que queremos que valga y despejemos $k$:


$f(a)^{\frac{log(k)+2log(a)}{log(k)+2log(b)}} < c \iff \frac{log(k)+2log(a)}{log(k)+2log(b)} < log_{f(a)}(c) = d \iff log(k)+2log(a) < d \cdot (log(k)+ 2log(b))$
$ \iff log(k\cdot a^2) < d \cdot log(k \cdot b^2) \iff k\cdot a^2 < (k\cdot b^2)^{d} \iff \sqrt[d-1]{\dfrac{a^2}{b^{2d}}} < k$
por lo tanto, si tomamos $k$ mayor a ese número horripilante, llegamos a la siguiente contradicción:

$c>f(a)^{\frac{log(k)+2log(a)}{log(k)+2log(b)}} \geq f(b)^{\frac{log(a)}{log(b)}}>c$

Como el absurdo provino de que existe tal constante $c$, concluímos que no existe y entonces probamos lo que queríamos.
Como $a$ y $b$ son arbitrarios, tenemos que vale la desigualdad simétrica $f(b)^{\frac{1}{log(b)}} \geq f(a)^{\frac{1}{log(a)}}$ y por lo tanto $f(a)^{\frac{1}{log(a)}} = f(b)^{\frac{1}{log(b)}}$ para todo par de enteros $a, b \geq 3$. Llamemos $C$ a este valor constante.

Tenemos que para todo $n \geq 3$ vale que $f(n)^{\frac{1}{log(n)}}=C$ con $C$ constante positiva, luego tenemos $f(n) = C^{log(n)} = n^{log(C)}=n^{\alpha}$ para todo $n\geq 3$ (notar que $\alpha = log(C) \geq 0$). Además $f(1)=f(1 \cdot 1)=f(1)^2$ por ser $1$ coprimo con sí mismo, luego $f(1)=1=1^{\alpha}$. Finalmente como $6^{\alpha} = f(6) = f(2)\cdot f(3) = f(2)\cdot 3^{\alpha}$ obtenemos que $f(2)=2^{\alpha}$.

Habiendo visto todos los casos, demostramos que $f(n)=n^{\alpha}$ para todo $n \in \mathbb{N}$ con $\alpha \geq 0$ constante, tal como queríamos. ■
2  
jujumas

OFO - Mención-OFO 2015 OFO - Medalla de Plata-OFO 2016 FOFO 6 años - Medalla Especial-FOFO 6 años OFO - Oro perfecto-OFO 2017 FOFO Pascua 2017 - Medalla-FOFO Pascua 2017
FOFO 7 años - Medalla Especial-FOFO 7 años OFO - Medalla de Oro-OFO 2018 FOFO 8 años - Jurado-FOFO 8 años OFO - Jurado-OFO 2019 FOFO Pascua 2019 - Jurado-FOFO Pascua 2019
FOFO 9 años - Jurado-FOFO 9 años OFO - Jurado-OFO 2020 COFFEE - Jurado-COFFEE Ariel Zylber
Mensajes: 402
Registrado: Dom 26 Oct, 2014 8:30 pm
Medallas: 13
Nivel: Exolímpico

Re: OFO 2018 Problema 10

Mensaje sin leer por jujumas »

Solución:
Spoiler: mostrar
Veamos primero que como $1$ y cualquier entero positivo $n$ son coprimos, reemplazando $m=1$ en la primera condición, tenemos que $f(1)f(n) = f(n)$, y como $f(n)$ es mayor a $0$ por definición, $f(1)=1$.



Lema 1: $f(x^an-x^a+1) \leq f(x)^a f(n) \leq f(x^an+x^a-1)$ para todos $x$ y $n$ enteros positivos coprimos mayores a $1$ y $a$ entero positivo.

Demostración:
Spoiler: mostrar
Vamos a demostrar ambos lados usando inducción en $a$.

Si $a=1$, la desigualdad se traduce a $f(xn-x+1) \leq f(x)f(n) \leq f(xn+x-1)$, y como $x$ y $n$ son coprimos, es equivalente a ver que $f(xn-x+1) \leq f(xn) \leq f(xn+x-1)$, lo que es trivial, ya que $x$ y $n$ son mayores a $1$ y la función es no decreciente.

Supongamos ahora que el lado izquierdo se cumple para $a=k$. Es decir, $f(x^kn-x^k+1) \leq f(x)^k f(n)$. Multiplicando ambos lados por $f(x)$, obtenemos que $f(x)f(x^kn-x^k+1) \leq f(x)^{k+1} f(n)$, pero notemos que si $d\mid x^kn-x^k+1$ y $d\mid x$, $d\mid x^kn-x^k+1 - (x^kn-x^k) = 1$, por lo que $d=1$ y $x$ y $x^kn-x^k+1$ son coprimos. Luego, como la función es multiplicativa para entradas coprimas, tenemos que la desigualdad es equivalente a $f(x(x^kn-x^k+1)) \leq f(x)^{k+1} f(n) \Rightarrow f(x^{k+1}n-x^{k+1}+x) \leq f(x)^{k+1} f(n)$, y como $x$ es mayor a $1$ y la función es no decreciente, tenemos que $(x^{k+1}n-x^{k+1}+1) \leq (x^{k+1}n-x^{k+1}+x) \leq f(x)^{k+1} f(n)$, lo que completa el paso inductivo.

(Se viene el copypaste)

Supongamos ahora que el lado derecho se cumple para $a=k$. Es decir, $f(x^kn+x^k-1) \geq f(x)^k f(n)$. Multiplicando ambos lados por $f(x)$, obtenemos que $f(x)f(x^kn+x^k-1) \geq f(x)^{k+1} f(n)$, pero notemos que si $d\mid x^kn+x^k-1$ y $d\mid x$, $d\mid x^kn+x^k-1 - (x^kn+x^k) = -1$, por lo que $d=1$ y $x$ y $x^kn-x^k+1$ son coprimos. Luego, como la función es multiplicativa para entradas coprimas, tenemos que la desigualdad es equivalente a $f(x(x^kn+x^k-1)) \geq f(x)^{k+1} f(n) \Rightarrow f(x^{k+1}n+x^{k+1}-x) \geq f(x)^{k+1} f(n)$, y como $x$ es mayor a $1$ y la función es no decreciente, tenemos que $(x^{k+1}n+x^{k+1}-1) \geq (x^{k+1}n+x^{k+1}-x) \geq f(x)^{k+1} f(n)$, lo que completa el paso inductivo.

Con ambas desigualdades demostradas, queda finalizada la demostración del Lema 1.


Lema 2: para todo $e$ real positivo, y enteros mayores a $1$ $a$ y $b$, existen enteros positivos $x$ e $y$ tales que $a^x \geq b^y$ y $\frac{a^x}{b^y} - 1 < e$

Demostración:
Spoiler: mostrar
Sumando $1$ a ambos lados, tenemos que $a^x \geq b^y$ y $\frac{a^x}{b^y} < e+1$. Notemos ahora que la función $log(x)$ es creciente al tomar entradas positivas, ya que si $w$ es mayor a $z$, $log(w) - log(z) = log(\frac{w}{z})$, y como $\frac{w}{z}$ es mayor a $1$, el valor tal que $10$ elevado a dicho valor sea igual a $\frac{w}{z}$ tiene que ser positivo.

Luego, como la función $log(x)$ es estrictamente creciente, tenemos que al tomar logaritmo de ambos lados de la desigualdad, lo que queremos probar es equivalente a $log(\frac{a^x}{b^y}) < log(e+1)$. Notemos sin embargo, que en el lado izquierdo tenemos que $log(\frac{a^x}{b^y}) = log(a^x) - log(b^y) = xlog(a) - ylog(b)$. Además, como $a^x \geq b^y$, tenemos que como $log$ es creciente, aplicando log a ambos lados tenemos que $xlog(a) \geq ylog(b)$. Luego, el Lema es equivalente a demostrar que dados $a$ y $b$ enteros positivos y $e$ real, podemos encontrar $x$ e $y$ enteros positivos tales que $0 \leq xlog(a) - ylog(b) < log(e+1)$. Notemos que al tender $e$ a $0$, como $log(1)=0$ y $log$ es creciente, $log(e+1)$ tiene a $0$. Luego, podemos demostrar que dado $r$ real positivo, podemos encontrar enteros positivos $x$ e $y$ tales que $0 \leq xlog(a) - ylog(b) < r$.

Notemos ahora que $log(a)$ y $log(b)$ son positivos, al ser $a$ y $b$ mayores a $1$. Luego, vamos a demostrar en general que dados reales positivos $u$, $v$ y $r$, existen enteros positivos $x$ e $y$ tales que $0 \leq xu - yv < r$. Esta propiedad es un teorema conocido, pero no me acuerdo su nombre, así que vamos a demostrarla:

Si $\frac{u}{v}$ es igual a un racional $\frac{p}{q}$, tomando $x=q$, $y=p$, obtenemos que $xu - yv=0$ y estamos. Supondremos entonces que $\frac{u}{v}$ es irracional. Vamos a demostrar primero que podemos lograr que $\left | xu - yv \right | < r$. Para ver esto, llamemos "resto módulo $v$" de un real como el resultado de restarle $v$ tantas veces a dicho real como sea posible sin que el resultado sea negativo. Claramente, dicho "resto" toma valores entre $0$ (inclusivo) y $v$ (exclusivo). Notemos que como la razón entre $u$ y $v$ es irracional, no habrá dos valores de $ku$ (donde $k$ es un entero positivo) con el mismo resto, ya que esto haría la razón entre $u$ y $v$ racional. Luego, la sucesión infinita $u$, $2u$, $3u$, $...$, toma infinitos restos, todos distintos, "módulo $v$", y como todos están en el rango entre $0$ y $v$, habrá necesariamente dos restos cuya diferencia será menor a $r$ (ya que de lo contrario sólo podría haber finitos restos).

Sean $nu$ y $mu$ los números con estos restos, con $n$ mayor a $m$, tenemos que $(n-m)u$ tiene o bien resto positivo menor a $e$, o resto negativo mayor a $-e$, por lo que tomando el valor de $y$ adecuado, podemos conseguir que $\left | xu - yv \right | < r$ con $x$ e $y$ positivos. De acá, tenemos dos casos. Si $0 < xu - yv < r$, tenemos lo pedido. Si $-r < xu - yv < 0$, lo que haremos será multiplicar $xu-yr$ por una constante positiva $k$ de modo que $-u< k(xu - yv) < -u+r$, cosa que claramente podemos lograr, ya que al multiplicar esta constante por un número suficientemente grande, el resultado tiende a menos infinito, y si ningún múltiplo positivo de $xu-yv$ entrara en este rango, tendríamos dos múltiplos consecutivos a distancia $r$ o mayor, cosa que sabemos que no puede pasar. Luego, tomando este múltiplo y sumando $u$ al resultado, obtenemos que $0 < kxu+ u - yv < r$, lo que termina de demostrar el Lema, ya que $x$ e $y$ son ambos positivos.


Con estos Lemas demostrados, vamos a tirar el problema abajo.

Como $f(1)=1$, $f(2)$ es al menos $1$. Si $f(2)=1$, podemos ver que $f(2)f(3)=f(6)$, por lo que $f(3)=f(6)$, y como $f$ es no decreciente, todos los valores de $f$ entre $3$ y $6$ son iguales. Podemos repetir esto con cada impar llegando a valores tan grande como queramos, lo que demostraría que todos los $f(n)$ son iguales a partir de $3$, pero como la función es multiplicativa en coprimos, tomando $n$ y $m$ coprimos mayores a $3$ en la primera condición, tenemos que la constante al cuadrado es la constante misma, y como esta es mayor a $0$, debe ser $1$. Luego, en este caso $f(n)=1$ para todo $n$.

Si $f(2)$ es mayor a $1$, podemos decir que $f(2)=2^{\alpha }$ con $\alpha$ real positivo, ya que $2^{\alpha }$ es sobreyectiva en los reales positivos para ${\alpha }$ positivo. Vamos a demostrar que si $f(m)=m^{\alpha }$, entonces $f(m+1)=(m+1)^{\alpha }$, lo que completaría la demostración por inducción. Para hacer esto, veremos dos casos.

Caso 1: $f(m)=m^{\alpha }$ y $f(m+1) < (m+1)^{\alpha }$.
Spoiler: mostrar
Vamos a demostrar que podemos encontrar enteros positivos $x$, $y$ y $n$ tales que se cumplan las desigualdades $(m+1)^xn-(m+1)^x \geq m^yn + m^y$ y $m^{\alpha }y > f(m+1)^x$. Para ver esto, notemos que la primera desigualdad es equivalente a $(m+1)^x(n-1) \geq m^y(n+1)$, o que $\frac{(m+1)^x}{m^y} \geq \frac{n+1}{n-1}$, mientras que la segunda es equivalente a $\frac{m^{\alpha }y}{f(m+1)^x} >1$, o que $\frac{f(m+1)^x}{m^{\alpha }y} < 1$. Como $f(m+1) < (m+1)^{\alpha }$, supongamos que $cf(m+1) = (m+1)^{\alpha }$, con $c$ mayor a $1$. Luego, multiplicando por $c^x$, la segunda desigualdad es equivalente a $\frac{cf(m+1)^x}{m^{\alpha }y} < c^x$, que es equivalente a $\frac{(m+1)^{\alpha }x}{m^{\alpha }y} < c^x$, que es equivalente a $\frac{(m+1)^x}{m^y} < c^{\frac{x}{\alpha}}$, ya que $x^{\alpha}$ es creciente en los reales positivos. Luego, lo que queremos se reduce a probar que podemos lograr que $c^{\frac{x}{\alpha}} > \frac{(m+1)^x}{m^y} \geq \frac{n+1}{n-1}$. Notemos entonces que por el Lema $2$, como $m+1$ y $m$ son coprimos y $m+1^x$ nunca será igual a $m^y$ para $m$ mayor a $1$, podemos reemplazando $c^{\frac{x}{\alpha}}$ por $e$, tenemos que podemos lograr que $c^{\frac{x}{\alpha}} > \frac{(m+1)^x}{m^y} > 1$. Luego, tomando $n$ suficientemente grande, $\frac{n+1}{n-1}$ puede acercarse tanto a $1$ como nosotros queramos, por lo que eventualmente será menor a $\frac{(m+1)^x}{m^y}$, lo que demuestra lo pedido.

Luego, usando el Lema 1, y los valores correctos de $x$ e $y$, tomando un valor de $n$ coprimo con $m$ y $m+1$ suficientemente grande (que existe, ya que $km(m+1)+1$ es coprimo con ambos para todo $k$) tenemos que $f(m+1)^xf(n) \geq f((m+1)^xn-(m+1)^x+1) \geq f((m+1)^xn-(m+1)^x)$ por el Lema $1$, que $f((m+1)^xn-(m+1)^x) \geq f(m^yn + m^y)$ al ser $f$ no decreciente, y que $f(m^yn + m^y) \geq f(m^yn + m^y - 1) \geq f(m)^yf(n)$ por el Lema 1. Pero sabemos que $f(m)^y = m^{\alpha }y > f(m+1)^x$. Luego, $f(m)^yf(n) > f(m+1)^xf(n)$, pero esto implica que $f(m+1)^xf(n) > f(m+1)^xf(n)$. Absurdo.
Caso 2: $f(m)=m^{\alpha }$ y $f(m+1) > (m+1)^{\alpha }$ (este caso es casi análogo al caso 1, así que muchos detalles van a ser pasados por arriba).
Spoiler: mostrar
De la misma forma que el caso anterior, podemos demostrar que podemos encontrar $x$, $y$ y $n$ suficientemente grande tales que se cumpla que $(m+1)^xn+(m+1)^x \leq m^yn - m^y \Rightarrow \frac{m^y}{(m+1)^x} \geq \frac{n+1}{n-1}$ y que se cumpla que $f(m+1)^x > m^{\alpha }y$, que es equivalente a que $\frac{f(m+1)^x}{m^{\alpha }y} > 1$, pero tomando $cf(m+1)=(m+1)^{\alpha }$, con $c$ menor a $1$, multiplicando por $c^x$, elevando a la $\frac{1}{{\alpha }}$ y luego tomando el inverso de ambos lados, obtenemos que $\frac{m^y}{(m+1)^x}$ tiene que ser menor a una constante mayor a $1$. Luego, por el Lema $2$, podemos lograr esto, y tomando $n$ suficientemente grande, demostramos la primer parte.

Ahora, la cadena de desigualdades en esta parte se vuelve $f(m+1)^xf(n) \leq f((m+1)^xn+(m+1)^x-1) \leq f((m+1)^xn+(m+1)^x) \leq f(m^yn - m^y) \leq f(m^yn - m^y + 1) \leq f(m)^yf(n)$, pero esto último es igual a $m^{\alpha}yf(n)$, que es menor a $f(m+1)^xf(n)$. Luego, $f(m+1)^xf(n) < f(m+1)^xf(n)$. Absurdo.
Luego, tenemos que $f(n)=n^{\alpha }$ para cierto ${\alpha }$ mayor o igual a $0$. Es facil demostrar que todas estas funciones son no decrecientes (creciente si ${\alpha }$ es mayor a $0$ y constante si es igual), y que $f(n)f(m)=n^{\alpha }m^{\alpha }=(nm)^{\alpha }=f(nm)$ para todos $m$ y $n$ coprimos.
3  
Avatar de Usuario
Elsa Muray

OFO - Medalla de Bronce-OFO 2018
Mensajes: 23
Registrado: Vie 26 Ene, 2018 3:10 pm
Medallas: 1
Nivel: 1
Ubicación: La Plata

Re: OFO 2018 Problema 10

Mensaje sin leer por Elsa Muray »

Dos curiosidades sobre el problema:

La primera:
Spoiler: mostrar
http://imomath.com/othercomp/Chn/ChnTST03.pdf
La segunda:
Spoiler: mostrar
https://drive.google.com/file/d/12MoTIX ... sp=sharing
(este apunte me lo pasó mi profe de olimpiadas, dijo que vio el teorema en una optativa y me mandó el archivo en PDF, pero que no utilizaba métodos habituales de ecuaciones funcionales que es lo que estuvimos practicando en las clases)
Espero con ansías leer una solución con conocimientos "elementales", cuando lo pensé no se me ocurrió nada :cry: :cry:
Avatar de Usuario
AgusBarreto

OFO - Medalla de Bronce-OFO 2015 OFO - Medalla de Bronce-OFO 2016 OFO - Jurado-OFO 2017 FOFO 7 años - Jurado-FOFO 7 años OFO - Jurado-OFO 2018
FOFO 8 años - Jurado-FOFO 8 años OFO - Jurado-OFO 2019 FOFO Pascua 2019 - Jurado-FOFO Pascua 2019 FOFO 9 años - Jurado-FOFO 9 años COFFEE - Jurado-COFFEE Matías Saucedo
OFO - Jurado-OFO 2020 FOFO Pascua 2020 - Jurado-FOFO Pascua 2020 COFFEE - Jurado-COFFEE Carolina González COFFEE - Jurado-COFFEE Ariel Zylber FOFO 10 años - Jurado-FOFO 10 años
OFO - Jurado-OFO 2021 FOFO 11 años - Jurado-FOFO 11 años OFO - Jurado-OFO 2022
Mensajes: 157
Registrado: Sab 15 Sep, 2012 6:28 pm
Medallas: 18
Nivel: Exolímpico
Ubicación: San Martín, Buenos Aires

Re: OFO 2018 Problema 10

Mensaje sin leer por AgusBarreto »

Elsa Muray escribió: Dom 04 Feb, 2018 11:29 pm Dos curiosidades sobre el problema:

La primera:
Spoiler: mostrar
http://imomath.com/othercomp/Chn/ChnTST03.pdf
La segunda:
Spoiler: mostrar
https://drive.google.com/file/d/12MoTIX ... sp=sharing
(este apunte me lo pasó mi profe de olimpiadas, dijo que vio el teorema en una optativa y me mandó el archivo en PDF, pero que no utilizaba métodos habituales de ecuaciones funcionales que es lo que estuvimos practicando en las clases)
Espero con ansías leer una solución con conocimientos "elementales", cuando lo pensé no se me ocurrió nada :cry: :cry:
@Elsa Muray espero haber saldado mi deuda, aunque no es tan elemental como me gustaría. Abrazo para tus profes que me caen muy bien.
2  
Responder